Đến nội dung

nam8298 nội dung

Có 158 mục bởi nam8298 (Tìm giới hạn từ 27-04-2020)



Sắp theo                Sắp xếp  

#458956 $\sum \frac{a-bc}{a+bc}\leq \fra...

Đã gửi bởi nam8298 on 20-10-2013 - 21:48 trong Bất đẳng thức và cực trị

Bài 3:Ta có $b^{2}c^{2}+a^{2}c^{2}\geq 2abc^{2}$   .suy ra  $\sum a^{2}b^{2}+2abc(a+b+c)\geq ab(ab+2c+2c^{2})$

suy ra $(ab+bc+ca)^{2}\geq ab(ab+2c+2c^{2})$  suy ra $\frac{1}{ab+2c+2c^{2}}\geq \frac{ab}{(ab+bc+ca)^{2}}$ tương tự rồi cộng vế




#459199 $max{a+c-b,b+c-a,c+a-b}\leq 1

Đã gửi bởi nam8298 on 22-10-2013 - 12:16 trong Bất đẳng thức - Cực trị

giả sử a=max{a,b,c} ta có (a+c-b)+(c+a-b) $\leq$ 2 nên a $\leq$ 1

do đó $0\leq a-bc\leq b-c+1-bc=(1-c)(1+b)$

          $0\leq a-bc\leq c-b+1-bc=(1-b)(1+c)$

nhân theo vế rồi rút gọn là xong




#459201 Tìm số k nhỏ nhất sao cho với mọi tập con A gồm k phần tử của X thì đều tồn t...

Đã gửi bởi nam8298 on 22-10-2013 - 12:38 trong Tổ hợp và rời rạc

Cho X là tập các số tự nhiên lẻ không chia hết cho 5 và nhỏ hơn 30 .Tìm số k nhỏ nhất sao cho với mọi tập con A gồm k phần tử của X thì đều tồn tại hai số trong A chia hết cho nhau




#459202 Chia tam giác đều cạnh $n$ thành $n^2$ tam giác đều cạnh...

Đã gửi bởi nam8298 on 22-10-2013 - 12:48 trong Tổ hợp và rời rạc

Bài 1 trong cuộc hội thảo cứ 10 người thì có đúng 1 người quen chung tìm số người quen lớn nhất của 1 người

Bài 2 Cho đa giác lồi n đỉnh sao cho không có 3 đường chéo nào đồng quy.tìm số miền do các đường chéo tạo nên

Bài 3 một tam giác đều n cạnh được chia làm $n^{2}$ tam giác đều cạnh 1 bằng các đường thẳng song song với các cạnh của nó .Hỏi có bao nhiêu tam giác đều được tạo thành

Bài 4 cho số nguyên $n\geq 2$ CMR trong mọi họ gồm ít nhất $2^{n-1}+1$ tập con không rỗng phân biệt của tập {1,2,3.....,n} đều tìm được 3 tập mà một trong chúng là hợp của 2 tập còn lại

 




#459643 CMR phương trình sau không có ngiệm nguyên dương $x^{4}-1=(2y+...

Đã gửi bởi nam8298 on 24-10-2013 - 15:53 trong Số học

CMR phương trình sau không có ngiệm nguyên dương $x^{4}-1=(2y+1)^{3}$




#459697 CMR phương trình sau không có ngiệm nguyên dương $x^{4}-1=(2y+...

Đã gửi bởi nam8298 on 24-10-2013 - 19:12 trong Số học

bạn ơi gcd(b+1,$b^{2}-b+1$) = 3 đc mà




#459869 .Tìm GTNN P =$(xy+yz+zx)^{2}-\frac{8}{(x+y...

Đã gửi bởi nam8298 on 25-10-2013 - 15:05 trong Bất đẳng thức - Cực trị

Cho x,y,z là các số thực thoả mãn $x^{2}+y^{2}+z^{2}=1$  .Tìm GTNN P =$(xy+yz+zx)^{2}-\frac{8}{(x+y+z)^{2}-xy-yz+2}$




#460039 Đề thi HSG lớp 12 tỉnh Vĩnh Phúc 2013-2014

Đã gửi bởi nam8298 on 26-10-2013 - 12:02 trong Thi HSG cấp Tỉnh, Thành phố. Olympic 30-4. Đề thi và kiểm tra đội tuyển các cấp.

1


Đăng lại đề @@
Bài 1. Cho trước số thực $a>0$ và dãy số thực $x_{n}$ xác định bởi $x_{1}$ =a và $x_{n+1}= \sqrt{17+16x_{n}}$ với mọi $n\geq 1$. Chứng minh rằng với mọi $a>0$ dãy $x_{n}$ có giới hạn khi $n\rightarrow$ dương vô cùng..Tìm giới hạn đó
Bài 2. Cho $3$ số $x,y,z$ không âm thỏa mãn $x^{2}+y^{2}+z^{2}= 1$ CMR $\sqrt{1-\frac{(x+y^{2})}{4}}+\sqrt{1-\frac{(y+z)^{2}}{4}}+\sqrt{1-\frac{(z+x)^{2}}{4}}\geq \sqrt{6}$
Bài 3. Tìm các số tự nhiên $x,y$ thỏa mãn phương trình ($(x^{2}+y)(y^{2}+x)= 2(x-y)^{3}$
Bài 4. Cho tam giác $ABC$ nhọn nội tiếp đường tròn $(O)$ vơí $AB<AC$ .Tiếp tuyến tại $A$ của $(O)$ cắt $BC$ tại $E$ . $D$ là điểm đối xứng của $A$ qua $O$,
a, Chứng minh rằng $AE$ song song với $CD$
b, Đường thẳng $BE$ cắt $AT$ tại $F$ .Giả sử đường tròn ngoại tiếp tam giác $AEF$ cắt $EO$ tại $G$ khác điểm $E$ .Chứng minh rằng tâm đường tròn nội tiếp tam giác $AGB$ nằm trên $(O)$
Bài 5. Một số nguyên dương $k$ được gọi là số đẹp nếu có thể phân hoạch tập hợp các số nguyên dương thành $k$ tập $A_{1},A_{2}....A_{k}$ sao cho với mỗi số nguyên dương $n\geq 15$ và với mọi i$\in (1;2;....:k)$ đều tồn tại 2 số thuộc $A_{i}$ có tổng là $n$
a, Chứng minh rằng $k=3$ là số đẹp
b. Chứng minh rằng với mọi $k\geq 4$ đều không đẹp.



#460344 Đề thi HSG lớp 12 tỉnh Vĩnh Phúc 2013-2014

Đã gửi bởi nam8298 on 27-10-2013 - 19:12 trong Thi HSG cấp Tỉnh, Thành phố. Olympic 30-4. Đề thi và kiểm tra đội tuyển các cấp.

xin lỗi mình đánh vội quá nên sai đề




#460919 $\frac{a^{3}}{b^{2}+c^{2}}+\frac{b^{3}}{c^{2}+a^{2}}+\fra...

Đã gửi bởi nam8298 on 30-10-2013 - 19:49 trong Bất đẳng thức - Cực trị

khẳng định bạn ạ




#460920 $a^3+b^3+c^3-3abc\geq 2\left ( \frac{b+c}{...

Đã gửi bởi nam8298 on 30-10-2013 - 19:59 trong Bất đẳng thức - Cực trị

+ nếu $\frac{b+c}{2}-a\leq 0$ ta đc đpcm

+nếu $\frac{b+c}{2}-a> 0$   đặt b=a+2x ; c=a+2y 

    đặt A= $a^{3}+b^{3}+c^{3}-3abc-2(\frac{b+c}{2}-a)^{3}$   suy ra A= $12a(x^{2}-xy+y^{2})+6(x+y)(x-y)^{2}\geq 6(x+y)(x-y)^{2}= \frac{3}{2}(\frac{b+c}{2}-a)(b-c)^{2}\geq 0$    suy ra BĐT đc cm




#460923 (a,b,c>0

Đã gửi bởi nam8298 on 30-10-2013 - 20:05 trong Bất đẳng thức - Cực trị

theo mình thì cái này hiển nhiên mà .....trong 3 số a,b,c có 2 số bằng nhau thì BĐT đc chứng minh

nếu không có 2 số nào bằng nhau  .khi đó trong các hiệu a-b ;b-c ;c-a có 1 số âm suy ra đpcm




#460925 Chứng minh $\sum \sqrt{1-\frac{(x+y^{2...

Đã gửi bởi nam8298 on 30-10-2013 - 20:11 trong Bất đẳng thức - Cực trị

bài này bình phương rồi dùng Cauchy-Schwazt




#460926 $\frac{bc}{3a^2+b^2+c^2}+\frac{ca...

Đã gửi bởi nam8298 on 30-10-2013 - 20:17 trong Bất đẳng thức - Cực trị

theo mình đánh giá thế này $\frac{bc}{3a^{2}+b^{2}+c^{2}}\leq \frac{(b+c)^{2}}{12a^{2}+2(b+c)^{2}}$   

chuẩn hóa a+b+c =3 .sau đó dùng ước lượng là đc




#461128 (a,b,c>0

Đã gửi bởi nam8298 on 31-10-2013 - 19:43 trong Bất đẳng thức - Cực trị

a=b=c=0




#461130 Cho a,b,c là các số thực không âm.CMR:

Đã gửi bởi nam8298 on 31-10-2013 - 19:47 trong Bất đẳng thức - Cực trị

a=0; b=c thay vào

 

Cho a,b,c là các số thực không âm .CMR: 

         $A=\frac{a^2}{b^2-bc+c^2}+\frac{b^2}{c^2-ac+c^2}+\frac{c^2}{a^2-ab+b^2}\geq 2$

 làm gì đúng




#461133 BĐT trê-bư-sép

Đã gửi bởi nam8298 on 31-10-2013 - 19:59 trong Bất đẳng thức - Cực trị

Cho a,b,c >0 .CMR $\frac{a}{b}+\frac{b}{c}+\frac{c}{a}\geq \sqrt{\frac{a^{2}+1}{b^{2}+1}}+\sqrt{\frac{b^{2}+1}{c^{2}+1}}+\sqrt{\frac{c^{2}+1}{a^{2}+1}}$




#461134 $\frac{a+b}{ab+a+b}+\frac{b+c}...

Đã gửi bởi nam8298 on 31-10-2013 - 20:02 trong Bất đẳng thức - Cực trị

Cho a,b,c >0 thỏa mãn $a^{2}+b^{2}+c^{2}= 1$  .CMR $\frac{a+b}{ab+1}+\frac{b+c}{bc+1}+\frac{c+a}{ca+1}\leq \frac{9}{2(a+b+c)}$




#462343 $\frac{1}{\sqrt{1+a^3}}+\fr...

Đã gửi bởi nam8298 on 05-11-2013 - 20:26 trong Bất đẳng thức và cực trị

1... $\sum \frac{bc}{\sqrt{a+bc}}= \sum \frac{bc}{\sqrt{(a+b)(a+c)}}\leq \frac{1}{2}\sum (\frac{bc}{a+b}+\frac{bc}{a+c})\leq \frac{1}{2}$




#462344 $\frac{1}{\sqrt{1+a^3}}+\fr...

Đã gửi bởi nam8298 on 05-11-2013 - 20:30 trong Bất đẳng thức và cực trị

2....a..$\frac{a^{3}}{(1+b)(1+c)}+\frac{1+b}{4}+\frac{1+c}{4}\geq 3a$

tương tự cộng theo vế suy ra ĐPCM

......b...$\sum \frac{1}{a^{3}(b+c)}= \sum \frac{(bc)^{2}}{ab+ac}\geq \frac{3}{2}$




#462348 $\frac{1}{\sqrt{1+a^3}}+\fr...

Đã gửi bởi nam8298 on 05-11-2013 - 20:36 trong Bất đẳng thức và cực trị

....3....$\frac{1}{\sqrt{1+a^{3}}}= \frac{1}{\sqrt{(1+a)(a^{2}-a+1)}} \geq \frac{2}{2+a^{2}}$

đến đây quy đồng là đc




#462558 Chứng minh rằng nếu A là tích của n số nguyên tố đầu tiên thì: p+1 và p-1 khô...

Đã gửi bởi nam8298 on 06-11-2013 - 21:02 trong Số học

 theo mình p là tích của n số nguyên tố đầu tiên .nếu thế mình chứng minh thế này

      p chia hết cho 3 nên p-1 chia 3 dư 2 nên không là số chính phương

      giả sử p+1 là số chính phương ..đặt p+1 =$a^{2}$ suy ra p =(a-1)(a+1) ..do p chẵn nên a lẻ .do đó a-1 và a+1 chẵn suy ra (a-1)(a+1) chia hết cho 4 suy ra p hia hết cho 4 (vô lí)

Vậy p-1 và p+1 không là số chính phương




#462746 Tìm GTLN của biểu thức $P=\frac{1}{2a+b+c}+...

Đã gửi bởi nam8298 on 07-11-2013 - 20:47 trong Bất đẳng thức và cực trị

chứng minh $\frac{1}{2a+b+c}\leq \frac{1}{4}(\frac{1}{a+b}+\frac{1}{a+c})\leq \frac{1}{16}(\frac{1}{2a}+\frac{1}{b}+\frac{1}{c})$

tương tự cộng vế




#463634 $n(n+1)(2n+1)\vdots 42$

Đã gửi bởi nam8298 on 11-11-2013 - 19:20 trong Đại số

bài 1 thay x =7k +3 ta đc (7k+3)(7k+4)(14k+7) cái này hiển nhiên là chia hết cho 42




#463636 $2x^2+x+3=3x\sqrt{x+3}$

Đã gửi bởi nam8298 on 11-11-2013 - 19:24 trong Phương trình - hệ phương trình - bất phương trình

đặt $\sqrt{x^{2}+7x+7}= a$ $3x^{2}+21x+18 = 3a^{2}-3$  ta đc phương trình bậc 2 có nghiệm là 1 và -5/3